PT2.S2.Q22 - despite improvements in treatment for asthma

SeriousbirdSeriousbird Alum Member
edited April 2016 in Logical Reasoning 1278 karma
I got this one right, and I was able to easily eliminate C-E, but I don't understand how to eliminate B.

I spent a lot of time debating between A and B. I chose A as the right answer because if urban populations have doubled in the past decade then it does not indicate an increase deaths but rather indicates the population was the same amount of deaths as before SO it is not an increased amount of deaths proportionate to the death rate.

Can someone please explain B to me!? Thanks!!!

Comments

  • runiggyrunruniggyrun Alum Inactive Sage Inactive ⭐
    2481 karma
    B helps the argument by chipping away at one of the alternative explanations: that asthma is now more widely and accurately recorded as a cause of death than it was before. B says that's not the case - the records were just as accurate 20 years ago. It's a very weak "strengthen" because it doesn't address the "widespread" part of the alternative explanation, but at least it clears up the fact that the accuracy is not an issue.
  • SeriousbirdSeriousbird Alum Member
    1278 karma
    Thanks! This really helps.. Added your explanation to my LR notebook!
  • knh9455knh9455 Alum Member
    23 karma

    So the AC B suggests that the using inhalers can be the cause of increase in death rate by asthma??
    I mean, the B supports that main conclusion?

  • AlpineDreamsAlpineDreams Alum Member
    edited December 2021 38 karma

    In a way, yes. The author is concluding that inhalers are the cause of the deaths, and B eliminates an alternate explanation: that the increased death rate is simply a result of statistical inaccuracy.

Sign In or Register to comment.